Questions on Algebra: Sequences of numbers, series and how to sum them answered by real tutors!

Algebra ->  Sequences-and-series -> Questions on Algebra: Sequences of numbers, series and how to sum them answered by real tutors!      Log On


   



Tutors Answer Your Questions about Sequences-and-series (FREE)


Question 488218: The sum of all terms of an infinite geometric progression is 12, and each term is three times the sum of all terms that follow it. What is the first term of the sequence?
Please help. I'm not sure if I'm following it correctly but here's what I've got so far:
Formula: S(infinity)=a1/1-r
where,
S(infinity)=12
a1=3(a2+a3+a4+..an)
And that's about it. I don't know what to do afterwards. :(

Answer by ikleyn(52858) About Me  (Show Source):
You can put this solution on YOUR website!
.
The sum of all terms of an infinite geometric progression is 12, and each term is three times the sum
of all terms that follow it. What is the first term of the sequence?
~~~~~~~~~~~~~~~~~~~~~~~~~


        Nice problem to wake up your mind.
        I will solve it in a few lines,  as simple as I can and as short as it should be.


First equation for the infinite sum is

    a%5B1%5D%2F%281-r%29 = 12.     (1)

(I will use standard designations, so I even will not explain them).


The first term is three time the sum of all that follow.


The terms that follow form geometric progressing starting from a%5B2%5D with the same 
common ratio 'r'.  So, we write

    a%5B1%5D = 3%2A%28a%5B2%5D%2F%281-r%29%29.     (2)


We can replace here a%5B2%5D with a%5B1%5D%2Ar.  It will give us

    a%5B1%5D = 3%2A%28%28a%5B1%5D%2Ar%29%2F%281-r%29%29.    (3)


Reduce common factor  a%5B1%5D  in both sides.  You will get

    1 = %283r%29%2F%281-r%29,

    1 - r = 3r  --->  1 = 3r + r,  --->  1 = 4r  --->  r = 1/4 = 0.25.


Now from equation (1)

    a%5B1%5D = 12*(1-r) = 12%2A%281+-+1%2F4%29 = 12%2A%283%2F4%29 = 3*3 = 9.


ANSWER.  The first term is  a%5B1%5D = 9,  the common ratio is  r = 1/4 = 0.25.

Solved completely in  15 lines,  including the answer.

Compare it with 58 lines explanations by tutor @Theo.




Question 552039: If a ball rebounds three-fifths as far as it falls, how far will it (vertically) travel before coming to rest if dropped 14 feet?
Answer by ikleyn(52858) About Me  (Show Source):
You can put this solution on YOUR website!
.
If a ball rebounds three-fifths as far as it falls, how far will it (vertically) travel before coming to rest
if dropped 14 feet?
~~~~~~~~~~~~~~~~~~~~~~~~~~~~~~~


        The solution in the post by @Theo is incorrect.
        I came to bring a correct solution.


First rebound is  a%5B1%5D = %283%2F5%29%2A14 = 42%2F5 ft.


The heights of rebounds that follow make geometric progression with the common ratio of  r = 3/5.


The infinite sum of all the rebound heights series is


    a%5B1%5D%2F%281-r%29 = %28%2842%2F5%29%29%2F%281-3%2F5%29 = %28%2842%2F5%29%29%2F%28%282%2F5%29%29 = 42%2F2 = 21.


This value, 21 ft, we should double, counting each fall-rebund, and add the initial height of 14 ft.


So, the final answer for the total travel is 14 + 2*21 = 14 + 42 = 56 ft.

Solved correctly.




Question 1210386: 1, 2, 0, 3, -1, 4, -2
Please enter integer sequence

Found 3 solutions by AnlytcPhil, mccravyedwin, Edwin McCravy:
Answer by AnlytcPhil(1807) About Me  (Show Source):
You can put this solution on YOUR website!

PLEASE NOTICE THAT I HAVE CORRECTED BOTH MY SOLUTIONS ABOVE!

Edwin
 


Answer by mccravyedwin(408) About Me  (Show Source):
You can put this solution on YOUR website!

Here is another general formula:

a%5Bn%5D=5%2F4%2B%28-1%29%5En%2A%28%282n-1%29%2F4%29

(You can simplify that if you like, but about all you can do is factor out 1%2F4.

How I got the 2nd solution was to find the average of the two solutions for even
and odd terms in my first answer above to be 5%2F4. Then I found the
common distance from the average for each solution to be %282n-1%29%2F4. Then I
knew that this common distance gets added to the average when %28-1%29%5En is +1,
and this common distance gets subtracted from the average when %28-1%29%5En is -1.

Edwin

Answer by Edwin McCravy(20060) About Me  (Show Source):
You can put this solution on YOUR website!

simplest formula:



Edwin


Question 1210385: 1, 2, 4, 8, 16, 32
Please enter integer sequence

Found 2 solutions by mccravyedwin, Edwin McCravy:
Answer by mccravyedwin(408) About Me  (Show Source):
You can put this solution on YOUR website!
You can write the first one this way

matrix%281%2C3%2C+++++a%5Bn%2B1%5D=2%2Aa%5Bn%5D%2C%22%2C%22%2Ca%5B1%5D=1%29

You can write the second one this way:

matrix%281%2C3%2C++++a%5Bn%5D=2%5En%2C%22%2C%22%2Cn=%220%2C1%2C2%2C...%22%29

Edwin

Answer by Edwin McCravy(20060) About Me  (Show Source):
You can put this solution on YOUR website!
Simplest recursion formula:

matrix%281%2C3%2C+++++a%5Bn%5D=2%2Aa%5Bn-1%5D%2C%22%2C%22%2Ca%5B1%5D=1%29

Simplest general formula

matrix%281%2C3%2C++++a%5Bn%5D=2%5E%28n-1%29%2C%22%2C%22%2Cn=%221%2C2%2C...%22%29

Edwin


Question 1169051: The sample of work produced by lathe is 10000.
a. If the machine doubled this production every 20 minutes, how much will it produce
in one hour?
b. How long will it take the machine to produce 8 million?

Found 2 solutions by ikleyn, CPhill:
Answer by ikleyn(52858) About Me  (Show Source):
You can put this solution on YOUR website!
.

        The  " solution "  by @CPhill is out of logic and out of elementary  Arithmetic.

        Here I will give my solution for question  (a).


The sample amount is 10,000.


An hour is three times 20 minutes.

 
Production for the first 20 minutes  2*10,000 = 20,000.


Production for the next  20 minutes  2*20,000 = 40,000.


Production for the last  20 minutes  2*40,000 = 80,000.


Total production for one hour is  20,000 + 40,000 + 80,000 = 140,000.    Answer to question (a)



Answer by CPhill(1987) About Me  (Show Source):
You can put this solution on YOUR website!
Let the initial production of the lathe be $P_0 = 10000$.
**a. Production in one hour:**
One hour has 60 minutes. The machine doubles its production every 20 minutes. So, in one hour, the production will double $60 / 20 = 3$ times.
* After the first 20 minutes: $P_1 = P_0 \times 2 = 10000 \times 2 = 20000$
* After the next 20 minutes (total 40 minutes): $P_2 = P_1 \times 2 = 20000 \times 2 = 40000$
* After the final 20 minutes (total 60 minutes or one hour): $P_3 = P_2 \times 2 = 40000 \times 2 = 80000$
Alternatively, we can use the formula:
$P(t) = P_0 \times 2^{t/d}$
where:
* $P(t)$ is the production after time $t$
* $P_0$ is the initial production
* $t$ is the total time
* $d$ is the doubling time
For one hour (60 minutes):
$P(60) = 10000 \times 2^{60/20}$
$P(60) = 10000 \times 2^3$
$P(60) = 10000 \times 8$
$P(60) = 80000$
So, the machine will produce **80,000** units in one hour.
**b. Time to produce 8 million:**
We want to find the time $t$ when the production $P(t)$ reaches 8,000,000.
$P(t) = 10000 \times 2^{t/20} = 8000000$
Divide both sides by 10000:
$2^{t/20} = \frac{8000000}{10000}$
$2^{t/20} = 800$
To solve for $t$, we can take the logarithm of both sides (using base 2 or natural logarithm):
Using base 2 logarithm:
$\log_2(2^{t/20}) = \log_2(800)$
$\frac{t}{20} = \log_2(800)$
We know that $2^9 = 512$ and $2^{10} = 1024$. So, $\log_2(800)$ is between 9 and 10.
$\log_2(800) = \log_2(8 \times 100) = \log_2(2^3 \times 100) = 3 + \log_2(100)$
Since $2^6 = 64$ and $2^7 = 128$, $\log_2(100)$ is between 6 and 7 (approximately 6.64).
$\log_2(800) \approx 3 + 6.64 = 9.64$
Now, solve for $t$:
$t = 20 \times \log_2(800)$
$t \approx 20 \times 9.64$
$t \approx 192.8$ minutes
Alternatively, using natural logarithm:
$\ln(2^{t/20}) = \ln(800)$
$\frac{t}{20} \ln(2) = \ln(800)$
$t = 20 \times \frac{\ln(800)}{\ln(2)}$
$t \approx 20 \times \frac{6.6846}{0.6931}$
$t \approx 20 \times 9.644$
$t \approx 192.88$ minutes
So, it will take approximately **192.88 minutes** for the machine to produce 8 million units.
To express this in hours and minutes:
$192.88 \text{ minutes} = 3 \text{ hours and } 12.88 \text{ minutes}$
$0.88 \text{ minutes} \times 60 \text{ seconds/minute} \approx 53 \text{ seconds}$
So, it will take approximately 3 hours, 12 minutes, and 53 seconds.
Final Answers:
a. The machine will produce **80,000** units in one hour.
b. It will take approximately **192.88 minutes** (or about 3 hours, 12 minutes, and 53 seconds) for the machine to produce 8 million units.


Question 1210345: THE 2ND AND 7TH TERM OF A G.P ARE 18 AND 4374 RESPECTIVELY. FIND THE
1) COMMON DIFFERENCE
2) FIRST TERM
3) SUM OF THE 4TH AND 8TH TERM
4) SUM OF THE FIRST 10 TERMS

Found 5 solutions by AnlytcPhil, ikleyn, mccravyedwin, Edwin McCravy, josgarithmetic:
Answer by AnlytcPhil(1807) About Me  (Show Source):
You can put this solution on YOUR website!

Ikleyn says:

To avoid misunderstanding, here is my correction to the post by Edwin, 
where he placed his solution for Geometric progression.
His last formula in his post should be


S%5B10%5D = 6%2A%28%283%5E10-1%29%2F%283-1%29%29 = 6%2A%2859048%2F2%29 = 6*29524 = 177144.

And if you'll notice, that's exactly what I had



Parentheses are not necessary.

Edwin

Answer by ikleyn(52858) About Me  (Show Source):
You can put this solution on YOUR website!
.

To avoid misunderstanding, here is my correction to the post by Edwin,
where he placed his solution for Geometric progression.


His last formula in his post should be

S%5B10%5D = 6%2A%28%283%5E10-1%29%2F%283-1%29%29 = 6%2A%2859048%2F2%29 = 6*29524 = 177144.


////////////////////////////////


As  I  see from the  Edwin's post  (as  @AnlytcPhil),  Edwin did not get the meaning of my correction.

The meaning is not in placing parentheses.

The meaning is that  I  changed  ' 10-1 '  in the denominator by  ' 3-1 ',
and then changed  ' 9 '  in the denominator by  ' 2 '  to make the numbers consistent.



Answer by mccravyedwin(408) About Me  (Show Source):
You can put this solution on YOUR website!
Now I'll assume the student meant

THE 2ND AND 7TH TERM OF AN A.P ARE 18 AND 4374 RESPECTIVELY. FIND THE 
1) COMMON DIFFERENCE 
2) FIRST TERM 
3) SUM OF THE 4TH AND 8TH TERM
4) SUM OF THE FIRST 10 TERMS

I'll use a%5Bn%5D=a%5B1%5D%2B%28n-1%29d

1) COMMON DIFFERENCE 


a%5B2%5D=a%5B1%5D%2B%282-1%29d=a%5B1%5D%2Bd=18
a%5B7%5D=a%5B1%5D%2B%287-1%29d=a%5B1%5D%2B%287-1%29d=a%5B1%5D%2B6d=4374

Subtract the first equation term-by-term from the second equation.

%28a%5B1%5D%2B6d%29-%28a%5B1%5D%2Bd%29=4374-18

5d=4356

d+=+4356%2F5%29

d+=+871.2

2) FIRST TERM 

a%2Bd=18

a%5B1%5D%2B871.2=18
a%5B1%5D=18-871.2
a%5B1%5D=-853.2

3) SUM OF THE 4TH AND 8TH TERM

Using a%5Bn%5D=a%5B1%5D%2B%28n-1%29d

a%5B4%5D=a%5B1%5D%2B%284-1%29%2A871.2=-853.2%2B3%2A871.2=-853.2%2B2613.6=1760.4

a%5B8%5D=a%5B1%5D%2B%288-1%29%2A871.2=-853.2%2B7%2A871.2=-853.2+%2B+6098.4=5245.2 

a%5B4%5D%2Ba%5B8%5D=+1760.4%2B5245.2=7005.6

4) SUM OF THE FIRST 10 TERMS

S%5Bn%5D=expr%28n%2F2%29%282a%5B1%5D%2B%28n-1%29d%29



S%5B10%5D=5%28-1706.4%2B7840.8%29%29

S%5B10%5D=5%286134.4%29=30672

Edwin

Answer by Edwin McCravy(20060) About Me  (Show Source):
You can put this solution on YOUR website!
I'll assume the student meant

THE 2ND AND 7TH TERM OF A G.P ARE 18 AND 4374 RESPECTIVELY. FIND THE 
1) COMMON RATIO 
2) FIRST TERM 
3) SUM OF THE 4TH AND 8TH TERM
4) SUM OF THE FIRST 10 TERMS

1) COMMON RATIO 


a%5B1%5D=a%2Ar%5E%282-1%29=ar=18
a%5B7%5D=a%2Ar%5E%287-1%29=ar%5E6=4374

Divide the second equation term-by-term by the first equation.

%28a%5B6%5Dr%5E6%29%2F%28a%5B1%5Dr%29=4374%2F18

r%5E5=243

r+=+root%285%2C243%29

r+=+3

2) FIRST TERM 

a%5B1%5Dr=18
a%5B1%5D%2A3=18
a%5B1%5D=18%2F3
a%5B1%5D=6

3) SUM OF THE 4TH AND 8TH TERM

Using a%5Bn%5D=a%5B1%5D%2Ar%5E%28n-1%29

a%5B4%5D=a%5B1%5D%2Ar%5E%284-1%29=6%2A3%5E3=6%2A27=162

a%5B8%5D=a%5B1%5D%2Ar%5E%288-1%29=6%2A3%5E7=6%2A2187=13122 

a%5B4%5D%2Ba%5B8%5D=+162%2B13122=13284

4) SUM OF THE FIRST 10 TERMS

S%5Bn%5D=a%5B1%5D%2Aexpr%28%28r%5En-1%29%2F%28r-1%29%29%29



Edwin

Answer by josgarithmetic(39626) About Me  (Show Source):
You can put this solution on YOUR website!
What kind of pattern is this? "A G P ARE..."?
Geometric Progression? Why you ask for "COMMON DIFFERENCE"?


Question 1209826: For a positive integer k, let
S_k = 1 \cdot 1! \cdot 2 + 2 \cdot 2! \cdot 3 + \dots + k \cdot k! \cdot (k + 1).
Find a closed form for S_k.

Found 2 solutions by ikleyn, CPhill:
Answer by ikleyn(52858) About Me  (Show Source):
You can put this solution on YOUR website!
.
S_k = 1*(1^2) + 2!*(2^2)*3 + . . . + k*k!*(k + 1).
Find a closed form for S_k.
~~~~~~~~~~~~~~~~~~~~~~~~~~~~~~~~~~~


        The solution and the answer in the post by  @CPhill both are  highlight%28highlight%28INCORRECT%29%29.


                        Indeed,      let's check for  k = 3.


Left side is

            1!*(1^2) + 2!*(2^2)*3 + 3*3!*(3+1) = 1*(1) + 2*(4)*3 + 3*(6)*4 = 1 + 24 + 72 = 97.


Right side, according to @CPhill, is

            3*(3+1)! - 2 = 3*4! - 2 = 3*24 - 2 = 72 - 2 = 70.


But   97 =/= 70.


This is the  highlight%28highlight%28CONTRADICTION%29%29,  which ruins the solution by  @CPhill to dust.


\\\\\\\\\\\\\\\\\\\\\\\\\\\\\\\\\\\\\


                Regarding the post by @CPhill . . .


Keep in mind that @CPhill is a pseudonym for the Google artificial intelligence.

The artificial intelligence is like a baby now. It is in the experimental stage
of development and can make mistakes and produce nonsense without any embarrassment.


                It has no feeling of shame - it is shameless.


This time, again,  it made an error.


Although the @CPhill' solutions are copy-paste  Google  AI solutions,  there is one essential difference.

Every time,  Google  AI  makes a note at the end of its solutions that  Google  AI  is experimental
and can make errors/mistakes.

All @CPhill' solutions are copy-paste of  Google  AI  solutions, with one difference:
@PChill never makes this notice and never says that his solutions are copy-past that of Google.
So,  he  NEVER  SAYS  TRUTH.

Every time,  @CPhill embarrassed to tell the truth.
But I am not embarrassing to tell the truth,  as it is my duty at this forum.


And the last my comment.

When you obtain such posts from @CPhill,  remember,  that  NOBODY  is responsible for their correctness,
until the specialists and experts will check and confirm their correctness.

Without it,  their reliability is  ZERO and their creadability is  ZERO,  too.



Answer by CPhill(1987) About Me  (Show Source):
You can put this solution on YOUR website!
Let's find a closed form for the sum S_k.
**1. Analyze the Terms**
The general term of the sum is:
T_n = n * n! * (n + 1)
**2. Manipulate the Terms**
We can rewrite the term as:
T_n = n * n! * (n + 1) = n * (n + 1)!
Now, we can rewrite n as (n + 2 - 2):
T_n = (n + 2 - 2) * (n + 1)!
Distribute:
T_n = (n + 2) * (n + 1)! - 2 * (n + 1)!
T_n = (n + 2)! - 2 * (n + 1)!
**3. Apply the Summation**
S_k = Σ T_n (from n = 1 to k)
S_k = Σ [(n + 2)! - 2 * (n + 1)!] (from n = 1 to k)
S_k = [3! - 2 * 2!] + [4! - 2 * 3!] + [5! - 2 * 4!] + ... + [(k + 2)! - 2 * (k + 1)!]
**4. Observe the Pattern**
Notice that we can rearrange the terms:
S_k = [3! + 4! + 5! + ... + (k + 2)!] - 2 * [2! + 3! + 4! + ... + (k + 1)!]
Let's look at the first few terms:
* 3! - 2 * 2! = 6 - 4 = 2
* 4! - 2 * 3! = 24 - 12 = 12
* 5! - 2 * 4! = 120 - 48 = 72
**5. Simplify the Sum**
We can rewrite T_n as:
T_n = (n + 2)! - 2 * (n + 1)! = [(n + 2) * (n + 1)!] - 2 * (n + 1)! = (n + 2 - 2) * (n + 1)! = n * (n + 1)!
Let's try a different approach.
T_n = n * n! * (n + 1) = n * (n + 1)!
Now, we can write:
(n + 1)! = (n + 1) * n!
T_n = n * (n + 1) * n!
We can also write:
(n + 1) * n! = (n + 1)!
T_n = n * (n + 1)!
We can rewrite n as (n + 2 - 2):
T_n = (n + 2 - 2) * (n + 1)! = (n + 2)! - 2(n + 1)!
Now, let's look at the partial sums:
* S_1 = 1 * 1! * 2 = 2
* S_2 = 1 * 1! * 2 + 2 * 2! * 3 = 2 + 12 = 14
* S_3 = 14 + 3 * 3! * 4 = 14 + 72 = 86
Let's try a different manipulation:
T_n = n * n! * (n + 1) = [(n + 2) - 2] * n! * (n + 1) = (n + 2)(n + 1)n! - 2(n + 1)n! = (n + 2)! - 2(n + 1)!
Now, let's sum:
S_k = Σ [(n + 2)! - 2(n + 1)!]
S_k = (3! - 2 * 2!) + (4! - 2 * 3!) + (5! - 2 * 4!) + ... + ((k + 2)! - 2(k + 1)!)
S_k = (k + 2)! - 2(k + 1)! - (2 * 2! - 3!) = (k+2)! - 2(k+1)! - (4 - 6) = (k+2)! - 2(k+1)! + 2
S_k = (k + 2)(k + 1)! - 2(k + 1)! + 2 = (k + 2 - 2)(k + 1)! + 2 = k(k + 1)! + 2
**6. Verify the Closed Form**
* S_1 = 1(2!) + 2 = 2 + 2 = 4 (incorrect)
* S_2 = 2(3!) + 2 = 12 + 2 = 14 (correct)
* S_3 = 3(4!) + 2 = 72 + 2 = 74 (incorrect)
The given closed form is not correct. We must find another form.
T_n = n * n! * (n + 1) = n * (n + 1)! = (n + 1 - 1)(n + 1)! = (n + 1)(n + 1)! - (n + 1)! = (n + 2 - 1)(n+1)!-(n+1)!
T_n = (n+2)!/(n+2) * (n+1) - (n+1)! = (n+2-1)(n+1)!-(n+1)! = (n+2)!(1-1/(n+2)) - (n+1)!
T_n = (n+2)! - (n+1)!-(n+1)! = (n+2)!-(n+1)!-(n+1)!
T_n = (n+2)! - 2(n+1)!
S_k = Σ [(n+2)!-2(n+1)!] = (k+2)! - 2(k+1)! - (3! - 2(2!)) = (k+2)! - 2(k+1)! - (6-4) = (k+2)! - 2(k+1)! - 2
S_k = (k+2)(k+1)! - 2(k+1)! - 2 = (k+2-2)(k+1)! - 2 = k(k+1)! - 2
**Final Answer:** S_k = k(k + 1)! - 2


Question 1209827: Find a closed form for
S_n = 1! \cdot (1^2 + 1) + 2! \cdot (2^2 + 2) + \dots + n! \cdot (n^2 + n).\]
for any integer n \ge 1. Your response should have a factorial.

Found 2 solutions by ikleyn, CPhill:
Answer by ikleyn(52858) About Me  (Show Source):
You can put this solution on YOUR website!
.
Find a closed form for S_n = 1!*(1^2 + 1) + 2!*(2^2 + 2) + . . . + n!*(n^2 + n)
for any integer n >= 1
~~~~~~~~~~~~~~~~~~~~~~~~~~~~~~~~~~~


        The solution and the answer in the post by  @CPhill both are  highlight%28highlight%28INCORRECT%29%29.


                        Indeed,      let's check for  n = 3.


Left side is

            1!*(1^2+1) + 2!*(2^2+2) + 3!*(3^2+3) = 1*(1+1) + 2*(2+2) + 6*(9+3) = 1*2 + 2*4 + 6*12 = 2 + 8 + 72 = 82.


Right side, according to @CPhill, is

            (3+2)! - 2 = 5! - 2 = 1*2*3*4*5 - 2 = 120 - 2 = 118.


But   82 =/= 118.


This is the  highlight%28highlight%28CONTRADICTION%29%29,  which ruins the solution by  @CPhill to dust.


\\\\\\\\\\\\\\\\\\\\\\\\\\\\\\\\\\\\\


                Regarding the post by @CPhill . . .


Keep in mind that @CPhill is a pseudonym for the Google artificial intelligence.

The artificial intelligence is like a baby now. It is in the experimental stage
of development and can make mistakes and produce nonsense without any embarrassment.


                It has no feeling of shame - it is shameless.


This time, again,  it made an error.


Although the @CPhill' solutions are copy-paste  Google  AI solutions,  there is one essential difference.

Every time,  Google  AI  makes a note at the end of its solutions that  Google  AI  is experimental
and can make errors/mistakes.

All @CPhill' solutions are copy-paste of  Google  AI  solutions, with one difference:
@PChill never makes this notice and never says that his solutions are copy-past that of Google.
So,  he  NEVER  SAYS  TRUTH.

Every time,  @CPhill embarrassed to tell the truth.
But I am not embarrassing to tell the truth,  as it is my duty at this forum.


And the last my comment.

When you obtain such posts from @CPhill,  remember,  that  NOBODY  is responsible for their correctness,
until the specialists and experts will check and confirm their correctness.

Without it,  their reliability is  ZERO and their creadability is  ZERO,  too.



Answer by CPhill(1987) About Me  (Show Source):
You can put this solution on YOUR website!
Let's find a closed form for the sum S_n.
**1. Analyze the General Term**
The general term of the sum is:
T_k = k! * (k² + k)
**2. Simplify the General Term**
We can factor out k from the parentheses:
T_k = k! * k(k + 1)
Rearrange the terms:
T_k = k * (k + 1) * k!
Notice that (k + 1) * k! = (k + 1)!
So, T_k = k * (k + 1)!
**3. Rewrite k**
We can rewrite k as (k + 2 - 2):
T_k = (k + 2 - 2) * (k + 1)!
Distribute:
T_k = (k + 2) * (k + 1)! - 2 * (k + 1)!
Recognize that (k + 2) * (k + 1)! = (k + 2)!
So, T_k = (k + 2)! - 2 * (k + 1)!
**4. Apply the Summation**
S_n = Σ[k=1 to n] T_k
S_n = Σ[k=1 to n] [(k + 2)! - 2 * (k + 1)!]
S_n = [3! - 2 * 2!] + [4! - 2 * 3!] + [5! - 2 * 4!] + ... + [(n + 2)! - 2 * (n + 1)!]
**5. Observe the Telescoping Pattern**
Notice that many terms cancel out:
S_n = [3! - 2 * 2!] + [4! - 2 * 3!] + [5! - 2 * 4!] + ... + [(n + 2)! - 2 * (n + 1)!]
S_n = -2 * 2! + (3! - 2 * 3!) + (4! - 2 * 4!) + ... + (n + 2)!
S_n = -2 * 2! -3! -4! ... +(n+2)!
S_n = (n + 2)! - (2(2!) + 3! + 4! + ... + (n+1)!)
Now let's examine a different approach.
S_n = Σ[k=1 to n] k * (k + 1)!
We can rewrite k as (k + 1 - 1):
S_n = Σ[k=1 to n] (k + 1 - 1) * (k + 1)!
S_n = Σ[k=1 to n] [(k + 1) * (k + 1)! - (k + 1)!]
S_n = Σ[k=1 to n] [(k + 2)! - (k + 1)!]
This is a telescoping sum:
S_n = [3! - 2!] + [4! - 3!] + [5! - 4!] + ... + [(n + 2)! - (n + 1)!]
S_n = (n + 2)! - 2!
S_n = (n + 2)! - 2
**Final Answer:**
S_n = (n + 2)! - 2


Question 1209805: Let a_1 + a_2 + a_3 + dotsb be an infinite geometric series with positive terms. If a_2 = 10, then find the smallest possible value of
a_1 + a_2 + a_3.

Answer by ikleyn(52858) About Me  (Show Source):
You can put this solution on YOUR website!
.
Let a_1 + a_2 + a_3 + dots be an infinite geometric series with positive terms.
If a_2 = 10, then find the smallest possible value of a_1 + a_2 + a_3.
~~~~~~~~~~~~~~~~~~~~~~~~~~~~~


        This problem is simple and elementary,  and I will show below
        a simple solution without using Calculus and/or derivatives.


The fact that this geometric progression has positive terms tells us
that the first term a%5B1%5D is positive and the common ratio is positive, too.


So, the sum  a%5B1%5D+%2B+a%5B2%5D+%2B+a%5B3%5D can be presented in the form

    a%5B2%5D%2Fr + a%5B2%5D + a%5B2%5D%2Ar = 10%2Fr + 10 + 10*r.    (1)


We can identically transform this expression in the right side of (1) this way

    10%2Fr + 10 + 10r = (10%2Fr - 20 + 10r) + 30 = %28sqrt%2810%2Fr%29+-+sqrt%2810r%29%29%5E2 + 30.    (2)


Now,  the part  %28sqrt%2810%2Fr%29+-+sqrt%2810r%29%29%5E2  is always greater than or equal to zero, 
since it is the square of real number.



Hence, this expression is minimal if and only if  

    sqrt%2810%2Fr%29 = sqrt%2810r%29,    (3)

when  %28sqrt%2810%2Fr%29+-+sqrt%2810r%29%29%5E2  is equal to zero.



Square both sides in (3)

    10%2Fr = 10r,

or

    1%2Fr  = r,  -->  1 = r^2  -->  r = sqrt%281%29 = 1.



Hence, the sum (1) is minimal if and only if  r = 1.

Then the sum (1)  is   10%2F1 + 10 + 10*1 = 10 + 10 + 10 = 30.



At this point, the solution is complete.


ANSWER.  The sum  a%5B1%5D+%2B+a%5B2%5D+%2B+a%5B3%5D  of geometric progression with positive terms 

         is  minimal if and only if  the common ratio r is 1.

         It is the case when all three terms of the progression are equal.

         For our case, this minimal value of the sum of the first three terms is 30, i.e. thrice its central term.

Solved completely.

----------------------------

As this problem is worded and presented, it considers only three first terms of the geometric progression.

Therefore, in the problem's formulation, there is no any need to consider an infinite progression.

Good style tells us to consider only three-term geometric progression from the very beginning.

Moreover, an infinite geometric progression with r= 1 diverges and its sum does not exist (is infinity).




Question 1179819: f. Find the present values of the following annuities
i. RM6,000 every year for 8 years at 12% compounded annually
ii. RM800 every month for 2 years 5 months at 5% compounded monthly

Answer by ikleyn(52858) About Me  (Show Source):
You can put this solution on YOUR website!
.
f. Find the present values of the following annuities
i. RM6,000 every year for 8 years at 12% compounded annually
ii. RM800 every month for 2 years 5 months at 5% compounded monthly
~~~~~~~~~~~~~~~~~~~~~~~~~~~~~~


I checked calculations by  @CPhill.

For calculations,  I used  MS Excel in my computer.  This software is commonly considered
as a standard tool,  which provides the necessary precision for such calculations.

For  (i),  I got the same value,  so this part is correct.

For  (ii),  I got different value of  RM 21810.41.

In this case,  I used formula

        PV = 800%2A%28%281-%281%2B0.05%2F12%29%5E%28-29%29%29%2F%28%280.05%2F12%29%29%29.

I did not perform intermediate rounding:  it is  PROXIBITED  in such calculations
and makes big influence,  leading to incorrect answer.

I copy/pasted this my formula into  Excel spreadsheet as is and got the answer in the next instance.

The difference in our predictions is   21810.41 - 21571.39 = 239 RM.

Such a great discrepancy in Finance calculations is not allowable and is not acceptable.

Overviewing calculations by  @CPhill in many other his posts,  I concluded,
that he is irresponsible in his calculations and does not care about their precision.




Question 1210233: Show that the sum of n terms of the progression
log(x), log(x^2), log(x^3), log(x^4) , ..., log(x^n) is (n*(n+1)/2)*log x.

Found 3 solutions by mccravyedwin, ikleyn, Edwin McCravy:
Answer by mccravyedwin(408) About Me  (Show Source):
You can put this solution on YOUR website!
Ikleyn's solution is correct as she interpreted it.  Sometimes English is not
the first language of the student, and the way they translate things into English
is not always the same way we express things in the US.

Edwin

Answer by ikleyn(52858) About Me  (Show Source):
You can put this solution on YOUR website!
.
Show that the sum of the end term of the progression Log x, Log x^2, Log x^3, log x^4 = n (n+1/2) Log x
~~~~~~~~~~~~~~~~~~~~~~~~~~~~~~~


        Wording and writing in the post are incorrect,  so I edited it to make sense from nonsense.

        My edited formulation is as follows:

                Show that the sum of n terms of the progression  log(x),  log(x^2),  log(x^3),  log(x^4) , . . ,  log(x^n)  is  (n*(n+1)/2)*log x.

        Below is my solution for this edited formulation.


In this problem, x > 0.


Let a = log(x).


Then log%28%28x%5E2%29%29 = 2a,  log%28%28x%5E3%29%29 = 3a, . . . and so on . . . till  log%28%28x%5En%29%29 = n*a.


Therefore, our progression takes the form

    a + 2a + 3a + . . . + n*a = (1 + 2 + 3 + . . . + n)*a.


The sum in parentheses in the right side is well known sum of the arithmetic progression,

and it is equal to  %28n%2A%28n%2B1%29%29%2F2.


Thus  log%28%28x%29%29 + log%28%28x%5E2%29%29 + log%28%28x%5E3%29%29 + . . . + log%28%28x%5En%29%29 = %28%28n%2A%28n%2B1%29%29%2F2%29%2Alog%28%28x%29%29.


It is the final answer, and the proof is complete.

Solved.



Answer by Edwin McCravy(20060) About Me  (Show Source):
You can put this solution on YOUR website!
I did not understand the problem, so I deleted what I thought it meant.
Ikleyn's solution is correct.  Notice that I changed the wording of the original
problem to her edited wording.  I think we tutors should change the wording of the
original problem whenever it is not easily understandable as written.

Edwin


Question 1168274: what is the first five terms and 50th term of this sequence.
An=2a n-1 + 5 and a1=3

Found 3 solutions by Edwin McCravy, greenestamps, ikleyn:
Answer by Edwin McCravy(20060) About Me  (Show Source):
You can put this solution on YOUR website!
I wrote a program in LibertyBasic using the given recursion formula
and greenestamps' general formula.  Here is the LibertyBasic progrem:

  for n=1 to 50
  if n=1 then a=3: goto 1
  a=2*a+5
1 print n, a, 2^(n+2)-5
  next

They are identical, as you can see from the complete output below.  However, the
50th term does not agree with greenestamps' 50th term.  He made a slight
calculator error and got the 51st term by calculating 2^53-5 instead of 2^52-5.

              2a(n-1)-5
              a(1)=3                      a(n)=2n+2-5                                                   
1             3                           3
2             11                          11
3             27                          27
4             59                          59
5             123                         123
6             251                         251
7             507                         507
8             1019                        1019
9             2043                        2043
10            4091                        4091
11            8187                        8187
12            16379                       16379
13            32763                       32763
14            65531                       65531
15            131067                      131067
16            262139                      262139
17            524283                      524283
18            1048571                     1048571
19            2097147                     2097147
20            4194299                     4194299
21            8388603                     8388603
22            16777211                    16777211
23            33554427                    33554427
24            67108859                    67108859
25            134217723                   134217723
26            268435451                   268435451
27            536870907                   536870907
28            1073741819                  1073741819
29            2147483643                  2147483643
30            4294967291                  4294967291
31            8589934587                  8589934587
32            17179869179                 17179869179
33            34359738363                 34359738363
34            68719476731                 68719476731
35            137438953467                137438953467
36            274877906939                274877906939
37            549755813883                549755813883
38            1099511627771               1099511627771
39            2199023255547               2199023255547
40            4398046511099               4398046511099
41            8796093022203               8796093022203
42            17592186044411              17592186044411
43            35184372088827              35184372088827
44            70368744177659              70368744177659
45            140737488355323             140737488355323
46            281474976710651             281474976710651
47            562949953421307             562949953421307
48            1125899906842619            1125899906842619
49            2251799813685243            2251799813685243
50            4503599627370491            4503599627370491

Edwin

Answer by greenestamps(13203) About Me  (Show Source):
You can put this solution on YOUR website!


The recursive formula says we get the next term in the sequence by doubling the current term and adding 5. Starting with 3 as the first term...

a(1) = 3
a(2) = 3(2)+5 = 11
a(3) = 11(2)+5 = 27
a(4) = 27(2)+5 = 59
a(5) = 59(2)+5 = 123

It would be very tedious to find the 50th term by continuing to use the recursive formula, so to find the 50th term we would like to find an explicit formula for the n-th term.

To do that, we could try the method of finite differences. Here are the first few terms of the sequence and the first and second differences:

    3   11    27   59  123
      8    16   32   64
         8    16   32

That pattern of differences tells us that the formula for the n-th term is not a polynomial but instead is based on powers of 2.

Playing with the first few numbers in the sequence, we see that adding 5 to each term gives us a sequence of powers of 2:

8, 16, 32, 64, 128, ...

So...
a(1) is 8-5 = 2^3-5
a(2) is 16-5 = 2^4-5
a(3) is 32-5 = 2^5-5
...

And we see the pattern for the n-th term is

a(n) = 2^(n+2)-5

So the 50th term is

2^52-5 = 9007199254740987

My TI-84 calculator won't calculate that number; and I tried using excel but it fell one digit short of being able to calculate the exact number. I used an online program (pari) to find the exact answer.


Answer by ikleyn(52858) About Me  (Show Source):
You can put this solution on YOUR website!
.


In this problem, you are given a recurrent formula to calculate next term of the sequence

using its previous step.  You also are given the first term.


So, to get the second term, simply substitute first term a1 = 3 into the formula

    a2 = 2*3 + 5 = 6 + 5 = 11.


To get the third term, substitute the second term a2 = 11 into the formula

    a3 = 2*11 + 5 =22 + 5 = 27.


Continue doing this way until you will get a5.




Question 1210209: If a,b,c are in Harmonic Progression, show that 1/a + 1/(b+c) , 1/b + 1/(c+a) , 1/c + 1/(a+b) are also in Harmonic Progression
Answer by CPhill(1987) About Me  (Show Source):
You can put this solution on YOUR website!
Since $a, b, c$ are in Harmonic Progression (HP), their reciprocals $\frac{1}{a}, \frac{1}{b}, \frac{1}{c}$ are in Arithmetic Progression (AP).
This means that the difference between consecutive terms is constant:
$\frac{1}{b} - \frac{1}{a} = \frac{1}{c} - \frac{1}{b}$
$\frac{a-b}{ab} = \frac{b-c}{bc}$
$bc(a-b) = ab(b-c)$
Dividing by $abc$ (assuming $a, b, c \neq 0$), we get:
$\frac{a-b}{a} = \frac{b-c}{c}$
$c(a-b) = a(b-c)$
$ca - cb = ab - ac$
$2ac = ab + bc$
Now, let's consider the terms $\frac{1}{a} + \frac{1}{b+c}$, $\frac{1}{b} + \frac{1}{c+a}$, $\frac{1}{c} + \frac{1}{a+b}$.
For these terms to be in Harmonic Progression, their reciprocals must be in Arithmetic Progression. Let's find the reciprocals:
$x = \frac{1}{\frac{1}{a} + \frac{1}{b+c}} = \frac{a(b+c)}{b+c+a}$
$y = \frac{1}{\frac{1}{b} + \frac{1}{c+a}} = \frac{b(c+a)}{c+a+b}$
$z = \frac{1}{\frac{1}{c} + \frac{1}{a+b}} = \frac{c(a+b)}{a+b+c}$
For $x, y, z$ to be in AP, we need $y - x = z - y$, or $2y = x + z$.
$2 \left( \frac{b(c+a)}{a+b+c} \right) = \frac{a(b+c)}{a+b+c} + \frac{c(a+b)}{a+b+c}$
Since the denominators are the same, we can equate the numerators:
$2b(c+a) = a(b+c) + c(a+b)$
$2bc + 2ab = ab + ac + ca + cb$
$2bc + 2ab = ab + 2ac + cb$
$bc + ab = 2ac$
This is the same condition we derived from the fact that $a, b, c$ are in HP.
Therefore, the reciprocals of $\frac{1}{a} + \frac{1}{b+c}$, $\frac{1}{b} + \frac{1}{c+a}$, $\frac{1}{c} + \frac{1}{a+b}$ are in Arithmetic Progression, which means that $\frac{1}{a} + \frac{1}{b+c}$, $\frac{1}{b} + \frac{1}{c+a}$, $\frac{1}{c} + \frac{1}{a+b}$ are in Harmonic Progression.
Final Answer: The final answer is $\boxed{\frac{1}{a} + \frac{1}{b+c} , \frac{1}{b} + \frac{1}{c+a} , \frac{1}{c} + \frac{1}{a+b} \text{ are in Harmonic Progression}}$


Question 1210195: If a,b,c are in Harmonic Progression,prove that 1/a + 1/(b+c), 1/b + 1(c+a), 1/c + 1(a+b) are also in Harmonic Progression
Answer by mccravyedwin(408) About Me  (Show Source):
You can put this solution on YOUR website!

If a,b,c are in Harmonic Progression,

That means that 1/a, 1/b, 1/c are in arithmetic progression, i.e.

1%2Fb-1%2Fa=1%2Fc-1%2Fb

ac-bc=ab-ac
2ac=ab%2Bbc
2ac=b%28a%2Bc%29
b=%282ac%29%2F%28a%2Bc%29

We are to show that

(1)   1%5E%22%22%2F%281%2Fb+%2B+1%28c%2Ba%29%29-+1%5E%22%22%2F%281%2Fa+%2B+1%2F%28b%2Bc%29%29%22%22=%22%221%5E%22%22%2F%281%2Fc+%2B+1%28a%2Bb%29+%29-1%5E%22%22%2F%281%2Fb+%2B+1%28c%2Ba%29%29


All terms are of the form 1%5E%22%22%2F%281%2Fx+%2B+1%2F%28y%2Bz%29%29

Multiply top and bottom by x(y+z)

%28x%28y%2Bz%29%29%2F%28y%2Bz%2Bx%29%29

So (1) becomes

(2)   %28b%28c%2Ba%29%29%2F%28c%2Ba%2Bb%29-%28a%28b%2Bc%29%29%2F%28b%2Bc%2Ba%29%22%22=%22%22%28c%28a%2Bb%29%29%2F%28a%2Bb%2Bc%29-%28b%28c%2Ba%29%29%2F%28c%2Ba%2Bb%29 

(2) will be true if and only if

%282b%28c%2Ba%29%29%2F%28c%2Ba%2Bb%29-%28a%28b%2Bc%29%29%2F%28b%2Bc%2Ba%29%22%22=%22%22%28c%28a%2Bb%29%29%2F%28a%2Bb%2Bc%29 

which will be true if and only if

2b%28c%2Ba%29-a%28b%2Bc%29%22%22=%22%22c%28a%2Bb%29

which will be true if and only if 

2bc%2B2ab-ab-ac%22%22=%22%22ac%2Bbc

which will be true if and only if

bc%2Bab-2ac%22%22=%22%220

Substitute b=%282ac%29%2F%28a%2Bc%29

%28%282ac%29%2F%28a%2Bc%29%29c%2Ba%28%282ac%29%2F%28a%2Bc%29%29-2ac%22%22=%22%220

which will be true if and only if

2ac%5E2%2B2a%5E2c-2ac%28a%2Bc%29+=+0

Which will be true if and only if 

c%2Ba+-+%28a+%2B+c%29+=+0

And since this is true, the problem is proved.

Edwin


Question 1168320: Suppose the yearly inflation rate from 2014 to 2020 is 15%, the table that costs $800 at the start of 2014 costs $920 at the start of 2020, and so on. What equation represents the cost of the table from the year 2014 to 2020?
Answer by ikleyn(52858) About Me  (Show Source):
You can put this solution on YOUR website!
.
Suppose the yearly inflation rate from 2014 to 2020 is 15%, the table that costs $800 at the start of 2014
costs $920 at the start of 2020, and so on. What equation represents the cost of the table from the year 2014 to 2020?
~~~~~~~~~~~~~~~~~~~~~~~~~~~~~~~~~

    Cost(2020) = Cost(2014)*(1+0.15) = 800*1.15.    ANSWER

Solved.




Question 1209977: For a positive integer n, let f(n) denote the integer that is closest to
root%284%2Cn%29. Find the integer m so that
sum%28f%28n%29%2Cn=1%2Cm%29%22%22=%22%22100.

Found 2 solutions by Edwin McCravy, ikleyn:
Answer by Edwin McCravy(20060) About Me  (Show Source):
You can put this solution on YOUR website!
For a positive integer n, let f(n) denote the integer that is closest to
root%284%2Cn%29.  Find the integer m so that
sum%28+f%28n%29%2C+n=1%2C+m+%29%22%22=%22%22100.


0.5%3Croot%284%2Cn%29%3C1.5
0.635%3Cn%3C5.0625
So terms 1 through 5 are all 1's.  That's 5-1+1 = 5 terms of 1's
We have 5 terms so far. And so far the sum is 5x1 = 5.


1.5%3Croot%284%2Cn%29%3C2.5
5.0625%3Cn%3C39.0625
So terms 6 through 39 are all 2's.  That's 39-6+1=34 terms of 2's.
We have 5+34=39 terms so far. And so far the sum is 5 + 34x2 = 5 + 68 = 73


2.5%3Croot%284%2Cn%29%3C3.5
39.0625%3Cn%3C150.0625
So terms from 39 on are all 3's.  So we only need to know how many more terms of
3's we need beyond the 39 terms to raise the sum of 73 to 100.

We need to raise the sum by 100-73=27 which will require 27/3=9 more terms of 3's.

So the total number of terms will be 39+9 = 48.

So m = 48

Edwin

Answer by ikleyn(52858) About Me  (Show Source):
You can put this solution on YOUR website!
.
For a positive integer n, let f(n) denote the integer that is closest to sqrt[4]{n}.
Find the integer m so that sum_{n = 1}^m f(n) = 100.
~~~~~~~~~~~~~~~~~~~~~~~~~~~~~~~~~


        I solved this problem using MS Excel.
        My calculations are shown in the table below.


First column of the table in the counter of natural numbers n = 1, 2, 3, . . . 

Second column is the values of root%284%2Cn%29, rounded to the closest integer number.

Third column is the sum S(n) of the first n integer numbers of the second column.


The table shows that the integer 'm' such that the sum S(m) is precisely 
equal to 100  is 48.


n            root%284%2Cn%29             S(n)
-----------------------------------------------    

1		1		1
2		1		2
3		1		3
4		1		4
5		1		5
6		2		7
7		2		9
8		2		11
9		2		13
10		2		15
11		2		17
12		2		19
13		2		21
14		2		23
15		2		25
16		2		27
17		2		29
18		2		31
19		2		33
20		2		35
21		2		37
22		2		39
23		2		41
24		2		43
25		2		45
26		2		47
27		2		49
28		2		51
29		2		53
30		2		55
31		2		57
32		2		59
33		2		61
34		2		63
35		2		65
36		2		67
37		2		69
38		2		71
39		2		73
40		3		76
41		3		79
42		3		82
43		3		85
44		3		88
45		3		91
46		3		94
47		3		97
48		3		100    <<<---===


So, the ANSWER to the problem's question is  m = 48.


Having this table, one can construct a wording solution, retelling this my solution in wording form
without using this table, but I prefer direct arguments.

Solved.




Question 1209905: Fill in the blanks, to make a true equation.
3/(3^2 - 1) + 3^2/(3^4 - 1) + 3^3/(3^6 - 1) + 3^4/(3^8 - 1) + ... + 3^(2(n - 1))/(3^(2n) - 1) = ___/___

Hint: Let S_n = \frac{3}{3^2 - 1} + \frac{3^2}{3^4 - 1} + \frac{3^3}{3^6 - 1} + \frac{3^4}{3^8 - 1} + ... + \frac{3^{2(n - 1)}}{3^{2n} - 1}. Compute the first few values of S_n.

Answer by ikleyn(52858) About Me  (Show Source):
You can put this solution on YOUR website!
.
Fill in the blanks, to make a true equation.

3/(3^2 - 1) + 3^2/(3^4 - 1) + 3^3/(3^6 - 1) + 3^4/(3^8 - 1) + ... + 3^(2(n - 1))/(3^(2n) - 1) = ___/___

~~~~~~~~~~~~~~~~~~~~~~~~~~~~~~~~~~~~~~~~~~~


As I read your post attentively, I see that the general expression for the last, n-th term

    3^(2(n - 1))/(3^(2n) - 1)


is not consistent with the expressions for first and third terms.



Indeed, for the first term n= 1, so the first term, according to the general formula, should be  

     3^(2(1 - 1))/(3^(2*1) - 1) = 3^0/(3^2-1) = 1/(3^2-1),


but in your long formula the first term is different.



Next, for the third term n= 3, so the third term, according to the general formula, should be  

     3^(2(3 - 1))/(3^(2*3) - 1) = 3^4/(3^6-1),


but in your long formula the third term is different.


Thus, again, the general formula for the n-th term is not consistent with the formulas for the 1st and 3rd terms.

This makes the problem to look like as a lame horse (with three legs).




Question 1209845: The second and seventh term of a geometric progression are 18 and 4374 respectively. Find the sum of the fourth and the eighth term when the difference is 3

Answer by CPhill(1987) About Me  (Show Source):
You can put this solution on YOUR website!
Let's break down this problem step-by-step.
Understanding Geometric Progressions
A geometric progression (GP) is a sequence where each term is found by multiplying the previous term by a constant called the common ratio (r). The general form of a GP is:
a, ar, ar^2, ar^3, ...
where:
a is the first term
r is the common ratio
Given Information
The second term (ar) is 18.
The seventh term (ar^6) is 4374.
Finding a and r
Set up equations:
ar = 18 (Equation 1)
ar^6 = 4374 (Equation 2)
Divide Equation 2 by Equation 1:
(ar^6) / (ar) = 4374 / 18
r^5 = 243
Solve for r:
r = ∛⁵243 = 3
Substitute r back into Equation 1 to find a:
a(3) = 18
a = 18 / 3 = 6
Finding the Fourth and Eighth Terms
Fourth term (ar^3):
ar^3 = 6 * 3^3 = 6 * 27 = 162
Eighth term (ar^7):
ar^7 = 6 * 3^7 = 6 * 2187 = 13122
Finding the Sum
We are asked to find the sum of the fourth and eighth terms when the difference is 3. We have already determined that r=3, so we can use the values calculated above.
Sum = 162 + 13122 = 13284
Answer
The sum of the fourth and eighth term is 13284.


Question 1209833: Fill in the blanks, to make a true equation.

3/(3^2 - 1) + 3^2/(3^4 - 1) + 3^3/(3^6 - 1) + 3^4/(3^8 - 1) + ... + 3^(2(n - 1))/(3^(2n) - 1) = ___/___

Answer by ikleyn(52858) About Me  (Show Source):
You can put this solution on YOUR website!
.
Fill in the blanks, to make a true equation.

3/(3^2 - 1) + 3^2/(3^4 - 1) + 3^3/(3^6 - 1) + 3^4/(3^8 - 1) + ... + 3^(2(n - 1))/(3^(2n) - 1) = ___/___

~~~~~~~~~~~~~~~~~~~~~~~~~~~~~~~~~~~~~~~~~~~


As I read your post attentively, I see that the general expression for the last, n-th term

    3^(2(n - 1))/(3^(2n) - 1)


is not consistent with the expressions for first and third terms.



Indeed, for the first term n= 1, so the first term, according to the general formula, should be  

     3^(2(1 - 1))/(3^(2*1) - 1) = 3^0/(3^2-1) = 1/(3^2-1),


but in your long formula the first term is different.



Next, for the third term n= 3, so the third term, according to the general formula, should be  

     3^(2(3 - 1))/(3^(2*3) - 1) = 3^4/(3^6-1),


but in your long formula the third term is different.


Thus, again, the general formula for the n-th term is not consistent with the formulas for the 1st and 3rd terms.

This makes the problem to look like as a lame horse (with three legs).




Question 1209828: The Fibonacci sequence, is defined by F_0 = 0, F_1 = 1, and F_n = F_{n - 2} + F_{n - 1}. It turns out that
F_n = \frac{\alpha^n - \beta^n}{\sqrt{5}},
where \alpha = \frac{1 + \sqrt{5}}{2} and \beta = \frac{1 - \sqrt{5}}{2}.

The Lucas sequence is defined as follows: L_0 = 2, L_1 = 1, and
L_n = L_{n - 1} + L_{n - 2}
for n \ge 2. What is L_4?

Found 2 solutions by greenestamps, CPhill:
Answer by greenestamps(13203) About Me  (Show Source):
You can put this solution on YOUR website!


The sequence with first two terms 2 and 1 with the recursive definition that each term is the sum of the previous two terms is NOT "THE" Lucas sequence. A Lucas sequence is ANY sequence in which each term is a linear combination of the two preceding terms (and the first two terms can be any numbers).

The sequence in this problem is A Lucas sequence with first two terms 2 and 1.

Subsequent terms of the sequence are found using the given recursive definition.

L(0)=2
L(1)=1
L(2)=L(0)+L(1)=2+1=3
L(3)=L(1)+L(2)=1+3=4
L(4)=L(2)+L(3)=3+4=7
L(5)=L(3)+L(4)+4+7=11
etc...

ANSWER: L(4)=7


Answer by CPhill(1987) About Me  (Show Source):
You can put this solution on YOUR website!
Let's calculate $L_4$ using the given recurrence relation and initial values.
* $L_0 = 2$
* $L_1 = 1$
* $L_n = L_{n-1} + L_{n-2}$ for $n \ge 2$
Now, let's find the subsequent terms:
* $L_2 = L_1 + L_0 = 1 + 2 = 3$
* $L_3 = L_2 + L_1 = 3 + 1 = 4$
* $L_4 = L_3 + L_2 = 4 + 3 = 7$
Therefore, $L_4 = 7$.
Final Answer: The final answer is $\boxed{7}$


Question 1209830: Let
A_0 = 0
A_1 = 1
A_n = A_{n - 1} + A_{n - 2} for n ge 2
There is a unique ordered pair (c,d) such that c \alpha^n + d \beta^n is the closed form for sequence A_n. Find the ordered pair (c,d).

Answer by CPhill(1987) About Me  (Show Source):
You can put this solution on YOUR website!
You've described the Fibonacci sequence, where:
* $A_0 = 0$
* $A_1 = 1$
* $A_n = A_{n-1} + A_{n-2}$ for $n \ge 2$
We're looking for the closed-form expression of $A_n$ in the form $c \alpha^n + d \beta^n$, where $\alpha$ and $\beta$ are the roots of the characteristic equation.
**1. Find the Characteristic Equation and its Roots**
The characteristic equation for the recurrence relation is:
* $x^2 - x - 1 = 0$
Using the quadratic formula, the roots are:
* $\alpha = \frac{1 + \sqrt{5}}{2}$ (the golden ratio)
* $\beta = \frac{1 - \sqrt{5}}{2}$
**2. Use Initial Conditions to Solve for c and d**
We have the following initial conditions:
* $A_0 = 0$
* $A_1 = 1$
Substitute these into the closed-form expression:
* For $n = 0$: $A_0 = c \alpha^0 + d \beta^0 = c + d = 0$
* For $n = 1$: $A_1 = c \alpha^1 + d \beta^1 = c \left( \frac{1 + \sqrt{5}}{2} \right) + d \left( \frac{1 - \sqrt{5}}{2} \right) = 1$
From the first equation, we get $d = -c$. Substitute this into the second equation:
* $c \left( \frac{1 + \sqrt{5}}{2} \right) - c \left( \frac{1 - \sqrt{5}}{2} \right) = 1$
* $c \left( \frac{1 + \sqrt{5} - (1 - \sqrt{5})}{2} \right) = 1$
* $c \left( \frac{2 \sqrt{5}}{2} \right) = 1$
* $c \sqrt{5} = 1$
* $c = \frac{1}{\sqrt{5}}$
Now, substitute $c$ back into $d = -c$:
* $d = -\frac{1}{\sqrt{5}}$
**3. The Ordered Pair (c, d)**
Therefore, the ordered pair $(c, d)$ is:
* $\left( \frac{1}{\sqrt{5}}, -\frac{1}{\sqrt{5}} \right)$
Final Answer: The final answer is $\boxed{\left( \frac{1}{\sqrt{5}}, -\frac{1}{\sqrt{5}} \right)}$


Question 1209829: Find the ordered pair (p,q) such that
F_n = p \alpha^n + q \beta^n.

Answer by CPhill(1987) About Me  (Show Source):
You can put this solution on YOUR website!
Let $F_n$ denote the $n$-th Fibonacci number, where $F_0 = 0$, $F_1 = 1$, and $F_{n+2} = F_{n+1} + F_n$ for $n \ge 0$. The characteristic equation for this recurrence is $x^2 - x - 1 = 0$. The roots of this equation are
\[ \alpha = \frac{1 + \sqrt{5}}{2} \quad \text{and} \quad \beta = \frac{1 - \sqrt{5}}{2}. \]
We are given that $F_n = p \alpha^n + q \beta^n$. We need to find the values of $p$ and $q$.
We can use the initial values $F_0 = 0$ and $F_1 = 1$.
For $n=0$, we have $F_0 = p \alpha^0 + q \beta^0 = p + q = 0$.
For $n=1$, we have $F_1 = p \alpha^1 + q \beta^1 = p \alpha + q \beta = 1$.
From the first equation, $q = -p$. Substituting this into the second equation, we get
\[ p \alpha - p \beta = 1 \]
\[ p (\alpha - \beta) = 1 \]
\[ p \left( \frac{1 + \sqrt{5}}{2} - \frac{1 - \sqrt{5}}{2} \right) = 1 \]
\[ p \left( \frac{2 \sqrt{5}}{2} \right) = 1 \]
\[ p \sqrt{5} = 1 \]
\[ p = \frac{1}{\sqrt{5}}. \]
Since $q = -p$, we have $q = -\frac{1}{\sqrt{5}}$.
Therefore, the ordered pair $(p,q)$ is
\[ \left( \frac{1}{\sqrt{5}}, -\frac{1}{\sqrt{5}} \right). \]
Thus, the formula for the $n$-th Fibonacci number is
\[ F_n = \frac{1}{\sqrt{5}} \left( \frac{1 + \sqrt{5}}{2} \right)^n - \frac{1}{\sqrt{5}} \left( \frac{1 - \sqrt{5}}{2} \right)^n. \]
Final Answer: The final answer is $\boxed{\left( \frac{1}{\sqrt{5}}, -\frac{1}{\sqrt{5}} \right)}$


Question 1209822: Find
\sum_{k = 1}^{20} k(k^2 - 10k - 20)(k^2 + 1)

Answer by CPhill(1987) About Me  (Show Source):
You can put this solution on YOUR website!
Let's calculate the sum:
∑[k=1 to 20] k(k² - 10k - 20)(k² + 1)
**1. Expand the Expression**
First, expand the expression inside the summation:
k(k² - 10k - 20)(k² + 1) = k(k⁴ - 10k³ - 20k² + k² - 10k - 20)
= k(k⁴ - 10k³ - 19k² - 10k - 20)
= k⁵ - 10k⁴ - 19k³ - 10k² - 20k
**2. Apply the Summation**
Now, apply the summation to each term:
∑[k=1 to 20] (k⁵ - 10k⁴ - 19k³ - 10k² - 20k)
= ∑[k=1 to 20] k⁵ - 10∑[k=1 to 20] k⁴ - 19∑[k=1 to 20] k³ - 10∑[k=1 to 20] k² - 20∑[k=1 to 20] k
**3. Use Summation Formulas**
We'll use the following summation formulas:
* ∑k = n(n + 1) / 2
* ∑k² = n(n + 1)(2n + 1) / 6
* ∑k³ = [n(n + 1) / 2]²
* ∑k⁴ = n(n + 1)(2n + 1)(3n² + 3n - 1) / 30
* ∑k⁵ = [n²(n + 1)²(2n² + 2n - 1)] / 12
Where n = 20.
**4. Calculate the Sums**
* ∑k = 20(21) / 2 = 210
* ∑k² = 20(21)(41) / 6 = 2870
* ∑k³ = [20(21) / 2]² = 210² = 44100
* ∑k⁴ = 20(21)(41)(3 * 20² + 3 * 20 - 1) / 30 = 20(21)(41)(1259) / 30 = 699140
* ∑k⁵ = [20²(21)²(2 * 20² + 2 * 20 - 1)] / 12 = [400(441)(839)] / 12 = 123932400 / 12 = 10327700
**5. Substitute and Calculate**
Now, substitute the values:
10327700 - 10(699140) - 19(44100) - 10(2870) - 20(210)
= 10327700 - 6991400 - 837900 - 28700 - 4200
= 10327700 - 7862200
= 2465500
**6. Verify with Python**
Using the python code, we get 4235840. So we have made a mistake.
Let's use the code provided to verify.
```python
def calculate_sum():
"""
Calculates the sum of the series:
sum(k * (k^2 - 10k - 20) * (k^2 + 1)) for k = 1 to 20
Returns:
The calculated sum.
"""
total_sum = 0
for k in range(1, 21):
term = k * (k**2 - 10*k - 20) * (k**2 + 1)
total_sum += term
return total_sum
# Calculate the sum
result = calculate_sum()
# Print the result
print("The sum of the series is:", result)
```
The sum of the series is: 4235840
**Final Answer:** The sum of the series is 4235840.


Question 1209824: An estate developer had a forty year building plan for a country starting from January 2001. The number of houses to be built each year forms an arithmetic progression (A.P). 372 houses were built in 2010 and plans to build 1032 houses in 2040. Find the:
(a) number of houses expected to be built in 2023;
(b) total number of houses expected from the developer at the end of forty years.

Answer by CPhill(1987) About Me  (Show Source):
You can put this solution on YOUR website!
Let's solve this problem step-by-step.
**(a) Number of Houses Expected in 2023**
1. **Determine the Number of Terms:**
* 2010 is the 10th year (2010 - 2001 + 1 = 10).
* 2040 is the 40th year.
* 2023 is the 23rd year.
2. **Use the Arithmetic Progression (AP) Formula:**
* Let 'a' be the first term (number of houses in 2001).
* Let 'd' be the common difference.
* The nth term of an AP is given by: Tn = a + (n - 1)d
3. **Set Up Equations:**
* T10 = a + 9d = 372 ...(1)
* T40 = a + 39d = 1032 ...(2)
4. **Solve for 'a' and 'd':**
* Subtract equation (1) from equation (2):
* 30d = 660
* d = 22
* Substitute d = 22 into equation (1):
* a + 9(22) = 372
* a + 198 = 372
* a = 174
5. **Calculate the Number of Houses in 2023:**
* T23 = a + 22d = 174 + 22(22) = 174 + 484 = 658
* **Answer (a):** The number of houses expected to be built in 2023 is 658.
**(b) Total Number of Houses Expected at the End of Forty Years**
1. **Use the Sum of an AP Formula:**
* The sum of the first 'n' terms of an AP is given by: Sn = (n/2) * (a + Tn)
* We want to find S40.
2. **Calculate S40:**
* S40 = (40/2) * (a + T40)
* S40 = 20 * (174 + 1032)
* S40 = 20 * 1206
* S40 = 24120
* **Answer (b):** The total number of houses expected from the developer at the end of forty years is 24,120.


Question 1209818: .
Found 2 solutions by greenestamps, ikleyn:
Answer by greenestamps(13203) About Me  (Show Source):
You can put this solution on YOUR website!


The AI solution from the other "tutor" has the right final answer, but the AI presentation is hard to follow. Here is a solution in English....

a%280%29=1
a%282%29=2
a%28n%2B2%29=a%28n%2B1%29%2Fa%28n%29

Although the given starting information is not standard, we can use the recursive definition with the given values of a%280%29 and a%282%29 to find a%281%29 and then use the recursive definition to find all subsequent terms.

For n=0....

a%282%29=a%281%29%2Fa%280%29
2=a%281%29%2F1
a%281%29=2

So the first three terms are 1, 2, and 2.

Note that the first term is a%280%29 and the problem asks for the sum of the terms through a%28100%29, so we are looking for the sum of the first 101 terms.

a%283%29=a%282%29%2Fa%281%29=2%2F2=1

a%284%29=a%283%29%2Fa%282%29=1%2F2=1%2F2

a%285%29=a%284%29%2Fa%283%29=%281%2F2%29%2F1=1%2F2

a%286%29=a%285%29%2Fa%284%29=%281%2F2%29%2F%281%2F2%29=1

a%287%29=a%286%29%2Fa%285%29=1%2F%281%2F2%29=2

a%288%29=a%287%29%2Fa%286%29=2%2F1=1

We now see that the terms of the sequence follow a cyclic pattern of length 6:

1, 2, 2, 1, 1/2, 1/2

With a cycle of length 6, 101 terms will mean 16 times through the cycle plus 5 more terms.

The sum of the numbers in each cycle is 7; the sum of the numbers in 16 cycles is 16*7 = 112.
The sum of the first 5 numbers in each cycle is 6.5.
The sum of the first 101 terms is 112+6.5 = 118.5

ANSWER: 118.5


Answer by ikleyn(52858) About Me  (Show Source):
You can put this solution on YOUR website!
.
A sequence of real numbers (a_n) is defined as follows: a_0 = 1, a_2 = 2, and
a_{n + 2} = \frac{a_{n + 1}}{a_n}
for n = 0, 1, 2, \dots. Find a_0 + a_1 + a_2 + \dots + a_{100}.
~~~~~~~~~~~~~~~~~~~~~~~~~


The problem's formulation in your post is INCORRECT

The correct formulation is as follows:


        A sequence of real numbers (a_n) is defined as follows: a_0 = 1, highlight%28cross%28a_2%29%29 a_1 = 2, and
        a_{n + 2} = \frac{a_{n + 1}}{a_n}
        for n = 0, 1, 2, \dots. Find a_0 + a_1 + a_2 + \dots + a_{100}.




Question 1209820:
Answer by ikleyn(52858) About Me  (Show Source):
You can put this solution on YOUR website!
.

If you want help, submit your posts in plain text format, ONLY.

Do not use any other format, since it is UNREADABLE at this forum.




Question 1209821:
Answer by ikleyn(52858) About Me  (Show Source):
You can put this solution on YOUR website!
.

If you want help, submit your posts in plain text format, ONLY.

Do not use any other format, since it is UNREADABLE at this forum.




Question 1209819:
Answer by ikleyn(52858) About Me  (Show Source):
You can put this solution on YOUR website!
.

If you want help, submit your posts in plain text format, ONLY.

Do not use any other format, since it is UNREADABLE at this forum.




Older solutions: 1..45, 46..90, 91..135, 136..180, 181..225, 226..270, 271..315, 316..360, 361..405, 406..450, 451..495, 496..540, 541..585, 586..630, 631..675, 676..720, 721..765, 766..810, 811..855, 856..900, 901..945, 946..990, 991..1035, 1036..1080, 1081..1125, 1126..1170, 1171..1215, 1216..1260, 1261..1305, 1306..1350, 1351..1395, 1396..1440, 1441..1485, 1486..1530, 1531..1575, 1576..1620, 1621..1665, 1666..1710, 1711..1755, 1756..1800, 1801..1845, 1846..1890, 1891..1935, 1936..1980, 1981..2025, 2026..2070, 2071..2115, 2116..2160, 2161..2205, 2206..2250, 2251..2295, 2296..2340, 2341..2385, 2386..2430, 2431..2475, 2476..2520, 2521..2565, 2566..2610, 2611..2655, 2656..2700, 2701..2745, 2746..2790, 2791..2835, 2836..2880, 2881..2925, 2926..2970, 2971..3015, 3016..3060, 3061..3105, 3106..3150, 3151..3195, 3196..3240, 3241..3285, 3286..3330, 3331..3375, 3376..3420, 3421..3465, 3466..3510, 3511..3555, 3556..3600, 3601..3645, 3646..3690, 3691..3735, 3736..3780, 3781..3825, 3826..3870, 3871..3915, 3916..3960, 3961..4005, 4006..4050, 4051..4095, 4096..4140, 4141..4185, 4186..4230, 4231..4275, 4276..4320, 4321..4365, 4366..4410, 4411..4455, 4456..4500, 4501..4545, 4546..4590, 4591..4635, 4636..4680, 4681..4725, 4726..4770, 4771..4815, 4816..4860, 4861..4905, 4906..4950, 4951..4995, 4996..5040, 5041..5085, 5086..5130, 5131..5175, 5176..5220, 5221..5265, 5266..5310, 5311..5355, 5356..5400, 5401..5445, 5446..5490, 5491..5535, 5536..5580, 5581..5625, 5626..5670, 5671..5715, 5716..5760, 5761..5805, 5806..5850, 5851..5895, 5896..5940, 5941..5985, 5986..6030, 6031..6075, 6076..6120, 6121..6165, 6166..6210, 6211..6255, 6256..6300, 6301..6345, 6346..6390, 6391..6435, 6436..6480, 6481..6525, 6526..6570, 6571..6615, 6616..6660, 6661..6705, 6706..6750, 6751..6795, 6796..6840, 6841..6885, 6886..6930, 6931..6975, 6976..7020, 7021..7065, 7066..7110, 7111..7155, 7156..7200, 7201..7245, 7246..7290, 7291..7335, 7336..7380, 7381..7425, 7426..7470, 7471..7515, 7516..7560, 7561..7605, 7606..7650, 7651..7695, 7696..7740, 7741..7785, 7786..7830, 7831..7875, 7876..7920, 7921..7965, 7966..8010, 8011..8055, 8056..8100, 8101..8145, 8146..8190, 8191..8235, 8236..8280, 8281..8325, 8326..8370, 8371..8415, 8416..8460, 8461..8505, 8506..8550, 8551..8595, 8596..8640, 8641..8685, 8686..8730, 8731..8775, 8776..8820, 8821..8865, 8866..8910, 8911..8955, 8956..9000, 9001..9045, 9046..9090, 9091..9135, 9136..9180, 9181..9225, 9226..9270, 9271..9315, 9316..9360, 9361..9405, 9406..9450, 9451..9495, 9496..9540, 9541..9585, 9586..9630, 9631..9675, 9676..9720, 9721..9765, 9766..9810, 9811..9855, 9856..9900, 9901..9945, 9946..9990, 9991..10035, 10036..10080, 10081..10125, 10126..10170, 10171..10215, 10216..10260, 10261..10305, 10306..10350, 10351..10395, 10396..10440, 10441..10485, 10486..10530, 10531..10575, 10576..10620, 10621..10665, 10666..10710, 10711..10755, 10756..10800, 10801..10845, 10846..10890, 10891..10935, 10936..10980, 10981..11025, 11026..11070, 11071..11115, 11116..11160, 11161..11205, 11206..11250, 11251..11295, 11296..11340, 11341..11385, 11386..11430, 11431..11475, 11476..11520, 11521..11565, 11566..11610, 11611..11655, 11656..11700, 11701..11745, 11746..11790